f(x)=-9x^2-2x and g(x)=-3x^2+6x-9, find (f-g)(x) and (f-g)(-4)

Answers

Answer 1

Answer:

(f - g)(x)= - 6 {x}^{2} - 8x + 9

(f - g)(-4!)= - 55

Step-by-step explanation:

[tex]f(x) = - 9 {x}^{2} - 2x, \: \: g(x) = - 3 {x}^{2} + 6x - 9 \\ (f - g)(x) = f(x) - g(x) \\ = - 9 {x}^{2} - 2x - (- 3 {x}^{2} + 6x - 9) \\ = - 9 {x}^{2} - 2x + 3 {x}^{2} - 6x + 9 \\ \purple{ \boxed{ \bold{(f - g)(x)= - 6 {x}^{2} - 8x + 9}}} \\ (f - g)( - 4)= - 6 {( -4 )}^{2} - 8( - 4) + 9 \\ = - 6 \times 16 + 32 + 9 \\ = - 96 + 41 \\ \red{ \boxed{ \bold{(f - g)( - 4)= - 55}}}[/tex]


Related Questions

Which equation has the solution x = 12?
(A
4x + 3 = 45
B
5x - 8= 68
2x - 5=29
3x +6=42

Answers

Answer:

D). last one

Step-by-step explanation:

a. 4x+3=45

4x=42

x=10.5 (incorrect)

b. 5x-8=68

5x=76

x=25.2 (incorrect)

c. 2x-5=29

2x=34

x=17 (incorrect)

d. 3x+6=42

3x=36

x=12

What’s the volume of the pic

Answers

Answer:

100[tex]\pi[/tex] cubic cm

Step-by-step explanation:

The volume of a cone can be represented by the formula [tex]\frac{1}{3} \pi r^{2} h[/tex]. Since we know the radius is 5, all we need to find id the height. To do that, we can use the Pythagorean theorem, and see that the height is equal to 12. Plugging the numbers into the formula gives us:

[tex]V =\frac{1}{3} \pi (5^{2}) (12)[/tex]

[tex]V = 100\pi[/tex]

HOPE THIS HELPED! :)

what is the cube root of 1

Answers

Answer:

1.

Step-by-step explanation:

∛1= 1.

It can also be seen as:

1×1×1= 1.

In ΔXYZ, the measure of ∠Z=90°, the measure of ∠X=57°, and XY = 8 feet. Find the length of YZ to the nearest tenth of a foot.

Answers

Answer:

21

Step-by-step explanation:

Answer:

6.7

Step-by-step explanation:

10 X 5/11=

just multiplication get it right and get brainlyest
60pts or whatever it narrows it down to

Answers

The answer is 4.55

When you divide 5 by 11 you will get  0.45454545454 than when you multiply these numbers by 10 you will get 4.54545454545 and then the rounded answer is 4.55.

Answer: 4.54

Step-by-step explanation:

A boat sailed 560 km in 8 hours. It took three hours to travel
the first 150 km. What was its average speed for the
remaining journey?
km/h​

Answers

Step-by-step explanation:

150 km in 3 hours

= 150 ÷ 3 = 50km/h

Remaining distance = 560 - 150 = 410

Remaining hours = 8 - 3 = 5

Then average speed = 410 ÷ 5 = 82 km / h

A company makes car batteries and claims 80% of its ABC batteries are good for 70 months or longer. Assume that this claim is true. Let p ˆ be the proportion in a sample of 100 such ABC batteries. What is the probability that this sample proportion is within 0.05 of the population proportion.

Answers

Answer:

78.88% probability that this sample proportion is within 0.05 of the population proportion

Step-by-step explanation:

We need to understand the normal probability distribution and the central limit theorem to solve this question.

Normal probability distribution

Problems of normally distributed samples are solved using the z-score formula.

In a set with mean [tex]\mu[/tex] and standard deviation [tex]\sigma[/tex], the zscore of a measure X is given by:

[tex]Z = \frac{X - \mu}{\sigma}[/tex]

The Z-score measures how many standard deviations the measure is from the mean. After finding the Z-score, we look at the z-score table and find the p-value associated with this z-score. This p-value is the probability that the value of the measure is smaller than X, that is, the percentile of X. Subtracting 1 by the pvalue, we get the probability that the value of the measure is greater than X.

Central Limit Theorem

The Central Limit Theorem estabilishes that, for a normally distributed random variable X, with mean [tex]\mu[/tex] and standard deviation [tex]\sigma[/tex], the sampling distribution of the sample means with size n can be approximated to a normal distribution with mean [tex]\mu[/tex] and standard deviation [tex]s = \frac{\sigma}{\sqrt{n}}[/tex].

For a skewed variable, the Central Limit Theorem can also be applied, as long as n is at least 30.

For proportion p in a sample of size n, we have that [tex]\mu = p, s = \sqrt{\frac{\pi(1-\pi)}{n}}[/tex]

In this question:

[tex]p = 0.8, n = 100[/tex]

So

[tex]\mu = 0.8, s = \sqrt{\frac{0.8*0.2}{100}} = 0.04[/tex]

What is the probability that this sample proportion is within 0.05 of the population proportion.

This is the pvalue of Z when X = 0.8 + 0.05 = 0.85 subtracted by the pvalue of Z when X = 0.8 - 0.05 = 0.75.

X = 0.85

[tex]Z = \frac{X - \mu}{\sigma}[/tex]

By the Central Limit Theorem

[tex]Z = \frac{X - \mu}{s}[/tex]

[tex]Z = \frac{0.85 - 0.8}{0.04}[/tex]

[tex]Z = 1.25[/tex]

[tex]Z = 1.25[/tex] has a pvalue of 0.8944.

X = 0.75

[tex]Z = \frac{X - \mu}{s}[/tex]

[tex]Z = \frac{0.75 - 0.8}{0.04}[/tex]

[tex]Z = -1.25[/tex]

[tex]Z = -1.25[/tex] has a pvalue of 0.1056.

0.8944 - 0.1056 = 0.7888

78.88% probability that this sample proportion is within 0.05 of the population proportion

will mark the branliest to first one who answers

Answers

Answer:

3 1/4

Step-by-step explanation:

3/4 + (1/3 ÷1/6) - (-1/2)

Subtracting a negative is adding

3/4 + (1/3 ÷1/6) +1/2

Parentheses first

Copy dot flip

3/4 + (1/3 * 6/1) +1/2

3/4 + 2 + 1/2

Get a common denominator

3/4 + 2 + 2/4

2 + 5/4

2 + 4/4 +1/4

2+1 + 1/4

3 1/4

the stained glass below shows bilateral symmetry. The two overlapping squares are congruent. What is the area of the window?

Answers

Answer:

  116.82 square inches

Step-by-step explanation:

The overall shape is that of a 10-inch square with four triangles attached. Each of those is an isosceles right triangle with leg lengths of 2.9 inches.

The area of the four triangles is ...

  total triangle area = 4(1/2)(2.9 in)(2.9 in) = 16.82 in²

The area of the 10-inch square is ...

  square area = (10 in)² = 100 in²

Then the total window area is ...

  window area = 16.82 in² +100 in²

  window area = 116.82 in²

Given: circle k(O), m AB = 120° m∠BCA = (x+60°) Find: x

Answers

Answer:

x=0

Step-by-step explanation:

<BCA is half of arc BA or half of 120 which is 60.

do the math, and then x has to be 0 degrees

Determine if the set of vectors shown to the right is a basis for IR3 If the set of vectors is not a basis, determine whether it is linearly independent and whether the set 311-4 spans R 12 Which of the following describe the set?
A. The set is a basis for R3
B. The set is linearly independent.
C The set spans R3
D. None of the above

Answers

Answer:

The problem is clearly solved in the attachment

Which line is parallel to y = 1/2x -5

Answers

Answer:

Any line with a slope of 1/2 would be parallel to that line.

Step-by-step explanation:

A line is parallel when the slopes are the same, causing the situation where the lines will never intercept. The line in the question has a slope of 1/2, so a parallel line must have that same slope.

The median and mode of this set of data (23,13,17,11,11)

Answers

Answer:

Mode: 11

Median: 13

Answer:

(23, 13, 17, 11, 11):

Median: 13

Arithmetic mean: 15

Geometric mean: 14.380735416546

Harmonic mean: 13.848764056076

Mode: 11

Standard deviation: 4.5607017003966

Variance: 20.8

Mean Absolute Deviation: 4

Range: 12

Interquartile range: 9

Lower quartile: 11

Upper quartile: 20

Quartile deviation: 4.5

Population size:5

perform the following operations on matrices (1 8 0 7) (7 6 7 4) = ( )

Answers

Answer:

  [tex]\left[\begin{array}{cc}63&38\\49&28\end{array}\right][/tex]

Step-by-step explanation:

  [tex]\left[\begin{array}{cc}1&8\\0&7\end{array}\right]\left[\begin{array}{cc}7&6\\7&4\end{array}\right]=\left[\begin{array}{cc}(1)(7)+(8)(7)&(1)(6)+(8)(4)\\(0)(7)+(7)(7)&(0)(6)+(7)(4)\end{array}\right]\\\\=\left[\begin{array}{cc}63&38\\49&28\end{array}\right][/tex]

Each element of the product matrix is the dot product of the corresponding row in the left matrix and the corresponding column in the right matrix.

For example, the element at row 2, column 1 of the product is [0 7]·[7, 7], the dot product of row 2 of the left matrix with column 1 of the right matrix.

_____

Many calculators, spreadsheets, and web sites can do this tedious math for you.

Junior bought a bag of mixed fruit snacks. The flavors in the bag are 4 strawberry, 3 cherry, and 5 grape. If he chooses one fruit snack at random, what it the probability of the first one being grape?

Answers

Answer:I believe it would be 5/12

Step-by-step explanation:

You add all of them up then since it's 5 grapes and in total there is 12 fruit snacks. It should be 5 grapes of 12 fruit snacks in the bag.

9x-3=87 what is the anwser

Answers

Answer:

x=10

Step-by-step explanation:

9x-3=87

add 3 to both sides

9x=90

divide by 9 on both sides

x=10

Answer:

x=10

Step-by-step explanation:

9x-3=87

you add 3 to both sides

9x-3(+3)=87(+3)

which equals

9x=90

90/9= 10

answer:

x =10

I hope this helped!

Stacy uses a spinner with six equal sections numbered 2, 2, 3, 4, 5, and 6 to play a game. Stacy spins the pointer 120 times and records the results. The pointer lands 30 times on a section numbered 2, 19 times on 3, 25 times on 4, 29 times on 5, and 17 times on 6.



Write a probability model for this experiment, and use the probability model to predict how many times Stacy would spin a 6 if she spun 50 times. Give the probabilities as decimals, rounded to 2 decimal places

Answers

Answer:

Hence, Stacy will spin 6,  8.33 times out of her n = 50 attempts.

Step-by-step explanation:

Let us consider a success to get a 6. In this case, note that the probability of having a 6 in one spin is 1/6. We can consider the number of 6's in 50 spins to be a binomial random variable. Then, let X to be the number of trials we get a 6 out of 50 trials. Then, we have the following model.

We will estimate the number of times that she spins a 6 as the expected value of this random variable.

Recall that if we have X as a binomial random variable of n trials with a probability of success of p, then it's expected value is np.

Then , in this case, with n=50 and p=1/6 we expect to have  number of times of having a 6, which is 8.33.

Please help ASAP! Will give BRAINLIEST! Please read the question THEN answer correctly! No guessing.

Answers

Answer:

B

Step-by-step explanation:

Answer:

B

Step-by-step explanation:

B because first you need to read the problem and understand the information.

Drag each tile to the correct box. Not all tiles will be used.
Arrange the equations in the correct sequence to find the inverse of f(x) = y = 3x / 8 + x​

Answers

Answer:

Inverse of f(x)

               [tex]f^{l} (x) = \frac{8 x}{3-x}[/tex]

Step-by-step explanation:

Explanation:-

Step(i):-

Given the function

                        [tex]f(x) = \frac{3 x}{8+x}[/tex]

Given function is one-one and onto function

Hence f(x) is bijection function

                   [tex]y = f(x) = \frac{3 x}{8+x}[/tex]

now cross multiplication, we get

            ( 8+x)y = 3 x

             8 y + x y = 3 x

             8 y = 3 x - x y

taking Common 'x' we get

            x (3 - y) = 8 y

                   [tex]x = \frac{8 y}{3-y}[/tex]

Step(ii):-

The inverse function

                 [tex]x = \frac{8 y}{3-y} = f^{l}(y)[/tex]

The inverse function of x

                  [tex]f^{l}(x) = \frac{8 x}{3-x}[/tex]

Final answer:-

Inverse of f(x)

               [tex]f^{l} (x) = \frac{8 x}{3-x}[/tex]

Sally wants to fill ten 8-inch tea glasses. How much tea does she need?
A) 80 ounces
B) 40 ounces
C) 80 cubic inches
D) Not enough information to answer

Answers

Answer

I believe C considering cubic mass

Answer:

D

Step-by-step explanation:

Suppose you had to
guess on a four-choice
multiple-choice test and
were given four questions.
Find the binomial
probability distribution.
( + ) ℎ =
4 = 0.25

Answers

Answer:

For 0 correct answer [tex]^4c_0p^0q^{4-0}[/tex]

For 1 correct answer [tex]^4c_1p^1q^{4-1}[/tex]

For 2 correct answer [tex]^4c_2p^0q^{4-2}[/tex]

For 3 correct answer [tex]^4c_3p^1q^{4-3}[/tex]

For 4 correct answer [tex]^4c_4p^1q^{4-4}[/tex]

Step-by-step explanation:

It is given that there are 4 questions n = 4

Number of choices is 4

So probability of getting correct answer [tex]=\frac{1}{4}[/tex]

Probability of getting incorrect answer [tex]=1-\frac{1}{4}=\frac{3}{4}[/tex]

Probability distribution is given by [tex]^nc_rp^rq^{n-r}[/tex]

Therefore probability distribution of 0 correct answer

[tex]^4c_0p^0q^{4-0}[/tex]

Therefore probability distribution of 1 correct answer

[tex]^4c_1p^1q^{4-1}[/tex]

Therefore probability distribution of 2 correct answer

[tex]^4c_2p^0q^{4-2}[/tex]

Therefore probability distribution of 3 correct answer.

[tex]^4c_3p^1q^{4-3}[/tex]

Therefore probability distribution of 4 correct answer.

[tex]^4c_4p^1q^{4-4}[/tex]

100 POINTS

PLEASE PROVIDE STEPS.

THANK YOU!!!

Answers

Answer:

⅓ m/s

Step-by-step explanation:

Area of a square is:

A = s²

Take derivative of both sides with respect to time:

dA/dt = 2s ds/dt

Given that dA/dt = 6 m²/s and s = 9 m:

6 m²/s = 2 (9 m) ds/dt

ds/dt = ⅓ m/s

The Formula of the area of a square: A = bh or A = s^2

Solution:

~Take derivative of both sides

da/dt = 2s * ds/dt

~Use given values (6m^2/s and 9m)

6 = 2(9) * ds/dt

~Simplify

1/3m/s = ds/dt

Best of Luck!

please ASAP , giving BRAINLIEST if correct.

Answers

Answer:

B. -3(4x + 1) (x - 4)

Step-by-step explanation:

Out of the other answer choices, "B," is the only that factorizes correctly and ends up with the correct factorization (It already gives you the break-down of the trinomial).

However, if you're unsure about the answer, you can always take the end result: -3(4x + 1) (x - 4), and multiply it together to see if you can end up with the original trinomial: [tex]-12x^2 + 45x + 12[/tex]

How many factors does 12 have

Answers

Answer:

6 if you count 1 and 12

Step-by-step explanation:

1*12

6*2

3*4

(1,12,3,4,6,2)

(12,1) (1,12) (3,4) (4,3) (6,2) (2,6)
2 negatives make a positive**
(-12,-1) (-1,-12) (-3,-4) (-4,-3) (-6,-2) (-2,6)

A slot machine has 3 dials each dial has 30 positions one of which is jackpot. To win jackpot all three dials must be in jackpot position. Assuming each play spins the dials and stops each independently and randomly, what are the odds of one play winning the jackpot

Answers

Answer:

3/90

Step-by-step explanation:

1 slot  is 1/30

2 slot is 1/30

3 slot is 1/30

this gives you that 3/90 when you had them

Answer:

D)  1/(30×30×30) = 1/27000 = 0.00003 or 0.003%

Step-by-step explanation:

which number when rounded to the nearest hundredth is 37.62

Answers

Answer:

Answer: B

Step-by-step explanation:

The number given is 37.62. So, by rounding to the nearest hundredths the number becomes 37.612 and this can be determined by using the rules of rounding off to the nearest hundredths.

Given :

Number - 37.62

The following steps can be used to round off any number to the nearest hundredth:

Step 1 - In the thousandths place value analyze that digit to round off a decimal.

Step 2 - If the value is 5 or more than 5 in the thousandth place then the digit is increased by 1 which is at the hundredth place and in the thousandth place digit and thereafter becomes zero.

Now, the number given is 37.62. So, by rounding to the nearest hundredths the number becomes 37.612.

Therefore, the correct option is B) 37.612.

For more information, refer to the link given below:

https://brainly.com/question/2096984

Which value has an absolute deviation of 5 from the mean of this data set?
26, 12, 35, 28, 14
A 28
B. 35
C. 26
D. 14

Answers

Answer: 28

Step-by-step explanation: see prev. explanation

The absolute deviation of 5 from the mean of this data set is 28.

What is absolute deviation?

Absolute deviation is "the distance between each data point to the mean".

According to the question,

The data set is 26, 12, 35, 28, 14

Average of the data set = [tex]\frac{sum of the data value }{Total number of observation}[/tex]

= [tex]\frac{26+12+35+28+14}{5}[/tex]

= [tex]\frac{115}{5}[/tex]

= 23.

Thus, the average of the data set is 23.

In order to find absolute deviation of 5 subtract each data point from the mean.

26 - 23 = |3| = 3

12 - 23  = |-11| = 11

35 - 23 = |12| = 12

28 - 23 = |5| =  5

14 - 23  = |-9| = 9.

Hence, the absolute deviation of 5 is from the mean of the data set is 28.

Learn more about absolute deviation here

https://brainly.com/question/4364130

#SPJ2

e is 5 more than d.
fis 7 less than d.
a) Write an expression for e in terms of d.

Answers

Answer:

C

Step-by-step explanation:

Answer:

e = d + 5

Step-by-step explanation:

e = d + 5

f = d - 7

Solve for e means write as e = ...d

and this is already there...

You can not write it more compact then this.

If you try, you will notice you finally end with the initial equation which you started with, or you endup with something which is obviously very true like

e = e or d = d.

What is the area of the triangle?
PLSSS help me

Answers

Answer:

The area of the triangle is [tex]A=6 \:units^2[/tex].

Step-by-step explanation:

The area A of a triangle is given by the formula [tex]A=\frac{1}{2} bh[/tex] where b is the base and h is the height of the triangle.

From the graph, we can see that the base is 3 units and the height is 4 units. Therefore, the area of the triangle is

[tex]A=\frac{1}{2} \cdot3\cdot 4=\frac{12}{2}=6 \:units^2[/tex]

Which statement is true?

Parallelograms are trapezoids.

A square is always a rhombus.

A rhombus is always a square.

A rectangle is a trapezoid.

Answers

Answer:

A square is always a rhombus.

Explanation:

A square has 4 sides of equal length.

Answer:

A square is always a rhombusStep-by-step explanation: trapezoids have 3 angles and 3 sides so in that case rectangles and parallelograms are not trapezoids.rhombuses aren't always squares.
Other Questions
Which of the following is NOT TRUE about Judaism. Women can be rabbis They worship in temples or synagogues They celebrate Easter Many Jews keep a kosher diet Today, there were 2 members absent from the band. The present members folded 25 programs each, for a total of 525 programs. How many members are in the band when no one is absent Help I have a very important question for the boys only? My teacher wants us to fo a project where we have to report what go's on in "the boys" group Provide one example of an adaption that a herbivore has to prevent itself from being eaten. Use a specific example. A plane intersects a pentagonal pyramid horizontally as shown. Describe the cross-section.A)pentagonB)rectangleC)trapezoidD)triangle A circular rug has the radius of 4 feet. Which of the following is closest to the number of square inches the rug covers.A. 55.26 ft2B. 50.24 ft2C. 29.7 ft2D. 33.6 ft2 Why did the Cold War become hot in Asia but not in Europe? Read the excerpt from the Declaration of Independence.We hold these Truths to be self-evident, that all Men are created equal, that they are endowed by their Creator with certain unalienable Rights, that among these are Life, Liberty, and the pursuit of Happiness. Which statement below best describes the language in this excerpt?1. objective and timid2. personal and casual3. complex and intellectual4. passionate and logical Prodigy math i will mark you brainliest if you get it right 1. Jayden spent $46.20 on 12 gallons of gasoline. What was the price per gallon? 2. Jayla buys 1.2 pounds of bananas. The bananas cost $1.29 per pound. To the nearest cent, how much does Jayla pay for the bananas? 3.Billy ate 1/4 of a pizza, and Wi ate 1/3 of the same pizza. How much of the Pizza did they eat in all?plz show the work a graduated cylinder has a mass of 50 g when empty. when 30 mL of water is added, the graduated cylinder has a mass of 120 g. if a rock is added to the grand cylinder, the water level rises to 75 mL and the total mass is now 250 g. what is the density of the rock? Simon is building a ramp in the shape of a triangular prism. He plans to paint each face of the ramp. What is the total surface area of the ramp?A triangular prism. The base has a length of 8 feet and height of 4 feet. A rectangular side has a base of 8 feet and height of 5 feet. Another rectangular side has a base of 8 feet and height of 3 feet. The triangular sides have a base of 4 feet and height of 3 feet. 68 square feet96 square feet108 square feet114 square feet A cube has a volume of 8 cubic feet. What is the length of the cube? In terms of forces, what makes the arrow start moving? Luke bought three canvas boxes to store his papers. The canvas boxes have dimensions of 12 inches by 10 inches by 12 inches. What volume of itmes can be the three canva boxes hold? The equation of a circle is (x2) 2 + (y6) 2 =64 . What is the center and radius of the circle? Explain why and how the Indians removed from their land and sent to Indian territory. What happened as a result of the Indian removal act? (1830) 30. Which of the following represents thestatement "The difference of two squarenumbers is positive A school dedicated 50% of the courtyard area for students to start a garden. The students want to know how much of the 632-square-foot space they will be able to use. Please help, need an answer ASAP!